valvulicous
opp's I did it again
(OPP Q's)
lets get in bed with beck
Dr. Anderson can check my ejection fraction anytime ;)
Kick-back induced arrhythmia's
100

Which of the following is most associated with the pathophysiology of pulmonary valve stenosis?


A. Left ventricular outflow obstruction

B. Diastolic heart failure

C. Wide-split S2 heart sound

D. Increased left ventricular pressure


C! wide split S2 heart sound

reason:

A wide-split S2 heart sound is associated with pulmonary valve stenosis. The second heart sound (S2) is formed by the closure of the aortic valve (A2) and the pulmonary valve (P2); in pulmonary stenosis, the P2 sound is delayed, causing a wide-split S2 sound. 

Contrast this with aortic stenosis, where the delayed closure of the aortic valve (A2) results in a narrow S2 split, creating a single S2 sound as the A2 and P2 sounds occur more closely together.

100

where is the anterior chapmans point for stomach acidity?

5th ICS on L

bonus- is this point best used for tx or dx?

100


Which of the following viruses is classically associated with pericarditis?

A. Rhinovirus
B. Parainfluenza virus
C. Coxsackievirus
D. Norovirus




C! Coxsackievirus

100


Which of the following is more likely to cause diastolic heart failure, instead of systolic heart failure?

A. Aortic stenosis
B. Ischemic heart disease
C. Chronical alcohol use
D. Viral myocarditis




A!

bonus points- what physiological proccess occurs with aortic stenosis that cuase diastolic HF? or what is normal systolic pressure in the LV?

100

Which of the following best describes the pathophysiology of atrial fibrillation?

a. ectopic pathway through bundle of kent

b. completelty independent and dissociated atrial and ventricular contractions

c. abberrant electrical foci originating pear pulmonary veins

d. re-entry circuit typically dounf in right atrium

C!

abberrant electrical foci originating pear pulmonary veins

200


The triad of exertional dyspnea, angina, and syncope is more suggestive of which valvular pathology?


A. Tricuspid regurgitation

B. Aortic stenosis

C. Mitral regurgitation

D. Mitral stenosis



B! Aortic stenosis

reason:

Exertional dyspnea occurs due to diastolic heart failure and the inability to fill up the left ventricle eventually leading to the backup of blood into the lungs. 


Angina occurs due to the lack of oxygen available for the cardiomyocytes due to the high myocardial demand placed on the heart because of the increased pressure needed to pump blood out of the left ventricle. This angina can occur in the absence of coronary artery disease.


Syncope occurs because less blood is able to be pumped out of the heart to perfuse organs, especially the brain. 

200

pt findings

- OA FRlSr

- T4 FRrSl

- L3 E RlSr

-L unilateral sacral flexionDoes this pt have a common, uncommon, or uncompensated compensatory patern?`

common compensated

200


Which of the following statements is true regarding the anatomy of the pericardium?


A. The visceral pericardium is the outermost layer of the pericardium

B. Pericardial fluid is located between the fibrous layer and the parietal layer

C. The fibrous pericardium is the innermost layer of the pericardium

D. The serous pericardium is composed of a parietal layer and a visceral layer



D!

bonus- whats another name for the visceral pericardium?

200

Which of the following conditions is unlikely to cause diastolic heart failure?


A. Mitral stenosis

B. Chronic hypertension

C. Aortic stenosis

D. Infiltrative cardiomyopathy





A!

bonus pts: what pathological process occurs due to mitral stenosis?

200

Auscultation of a patient reveals a classic mid-diastolic murmur with an opening snap. Which of the following statements describes this disease process?

a. revesal of blood flow from aorta to LV

b. obstruction of blood flow from left atrium to LV

c. reversal from LV to LA

d. obstruction of blood floe from LV to Aorta

B!

obstruction of blood flow from left atrium to LV

300


Which of the following pathophysiological findings is not associated with chronic aortic regurgitation?


A. Systolic heart failure

B. Aortic dissection

C. Left ventricular eccentric hypertrophy

D. S3 heart sound

E. Increased pulse pressure


B! aortic dissection

this would be acute aortic regurg

300

If stimulation at t5 spinal segment causes discharge at t3 spinal segment, what is the T3 spinal segment called and what is going on to the interneurons at T3 spinal segemnt?

t3 is the fascilitated segment, interneurons are hyperexciatble or functioning at a lower threshold.
300

Constrictive pericarditis is not associated with which of the following clinical features?

A. Jugular venous distension

B. Hepatomegaly

C. Bradycardia

D. Peripheral edema


C! Bradycardia

bonus points--> why?

300


Which of the following are associated with systolic heart failure and diastolic heart failure, respectively?


A. Reduced ejection fraction; increased ejection fraction

B. Eccentric hypertrophy; concentric hypertrophy

C. Increased ejection fraction; preserved ejection fraction

D. Concentric hypertrophy; eccentric hypertrophy



B!

reason:

In systolic heart failure, there is eccentric hypertrophy because contractile proteins are being added in series, creating a dilated ventricle with low contractility. In diastolic heart failure, there is concentric hypertrophy because contractile proteins are being added in parallel, creating a stiff, thickened (and restrictive) ventricle with low compliance.

300

Why might digoxin be useful in treating atrial fibrillation secondary to systolic heart failure?

a. digoxin increases vagal tone and is a positive ionotrope

b. digoxin decreases vagal tone and is a neg. ionotrope

c. its increases vagal tone and is a neg ionotrope

d. it decreases vagal tone and is a positive ionotrope

A!

. digoxin increases vagal tone and is a positive ionotrope

400


Which of the following does not cause aortic dilatation and subsequently, lead to aortic regurgitation?


A. Aortic dissection

B. Connective tissue disease

C. Tertiary syphilis

D. Chronic, poorly controlled hypertension

E. Rheumatic fever


E! rheumatic fever

reason:

Rheumatic fever is the most common cause of mitral stenosis. Mitral stenosis caused by rheumatic fever can occur years after infection. Although rheumatic fever is associated with aortic regurgitation, it is not associated with aortic dilatation causing aortic regurgitation.


Rheumatic fever is most often caused by an untreated group A β-hemolytic streptococcus bacterial infection (most often S. pyogenes) which also causes strep throat (pharyngitis).


400

do asymmetric organs cause ipsilateral or contralateral  reflexes?

ipsilateral

400


Which of the following is not a feature of Beck's triad?

A. Hypotension
B. Pulsus paradoxus
C. Muffled heart sounds
D. Jugular venous distension


B! pulsus paradoxus

bonus- what vavlular disease is this exam finding a sign of?

400


Which of the following is a classic cause of low-output heart failure?

A. Hyperthyroidism

B. Thiamine deficiency

C. Severe anemia

D. Cardiogenic shock

E. Arteriovenous fistula



D!


bonus- why does thiamine def or severe anemia cause high out put HF?

400


Which of the following drugs is not used for rate control in atrial fibrillation?

A. Metoprolol
B. Verapamil
C. Digoxin
D. Quinidine




B! 

verapamil

bonus points- what class antiarrhythmic is verapamil?

500

A previously healthy 62-year-old female presents to a primary care clinic to establish care. She has never taken any medications. She was last seen by a doctor over 15 years ago and has not kept up with health maintenance screenings. The patient is happy to report that she has lost 20 lbs (9.1 kg) in the last four months without modifying her diet or exercise. She denies fevers, chills, bruising, bleeding, or recent sick contacts but notes mild fatigue. Vital signs include a temperature of 38.5°C (101.3°F), blood pressure of 145/80 mmHg, heart rate of 80 beats/min, and respiratory rate of 18 breaths/min. Physical exam reveals conjunctival pallor and a holosystolic murmur best heard at the cardiac apex. This patient’s current presentation is likely a consequence of which of the following?


A. Bacterial infection

B. Hypocoagulable state

C. Intravenous drug use

D. Recent dental procedure

E. Underlying malignancy



E!

This patient's unintentional 20 lbs (9.1 kg) weight loss over four months without changes in diet or exercise habits is concerning for an underlying malignancy. Fatigue and fever may also be associated with cancer. This patient's conjunctival pallor is likely due to anemia of chronic disease, which may occur in the setting of malignancy.


The presence of a holosystolic murmur in this patient could be related to nonbacterial thrombotic endocarditis (NBTE). NBTE is a form of endocarditis that is not caused by bacterial infection but rather by the formation of sterile vegetations on the heart valves. 


This condition is often associated with underlying malignancies (particularly adenocarcinomas) and other hypercoagulable states (e.g., systemic lupus erythematosus and antiphospholipid syndrome). The management of NBTE involves anticoagulation therapy and monitoring for potential embolic complications.

500

what ganglion is associated with vertebral levels T5-9 and the foregut?

celiac

500

A 76-year-old male presents to the emergency department with new-onset chest pain and shortness of breath. The patient is status post cardiac catheterization and stent placement after having an anterior wall ST-segment elevation myocardial infarction one week prior. Vital signs include a blood pressure of 80/50 mmHg, heart rate of 110 beats/min, and respiratory rate of 18 breaths/min. Jugular venous pressure is 15 cm of H2O on physical exam. No murmurs, rubs, or gallops are noted. On inhalation, the patient's systolic blood pressure decreases by 22-26 mmHg. His chest x-ray is shown below. What is the most likely cause of this patient’s presentation?



A. Dressler syndrom

B. Left ventricular free-wall rupture

C. Left ventricular pseudoaneurysm formation

D. Papillary muscle rupture

E. Ventricular septal rupture

B!

reason:
This patient presenting one week following an anterior wall myocardial infarction with hypotension, jugular venous distention, and cardiomegaly on chest x-ray likely has a left ventricular free wall rupture.


Left ventricular free wall rupture is the most common form of myocardial rupture following a myocardial infarction (MI). It usually occurs early in the recovery process due to limited collagen deposition at the infarct site, most commonly the anterolateral wall, leaving the tissue vulnerable. These ruptures allow blood to leak into the pericardial sac, causing hemopericardium, potentially compromising cardiac outflow due to cardiac tamponade. 


Tamponade leads to hypotension and jugular venous distention, as seen in this patient. A chest x-ray will classically reveal a "water-bottle" heart sign. Blood pressure monitoring can show pulsus paradoxus (a decrease in systolic blood pressure > 10 mmHg during inspiration)

500

A 19-year-old female presents to the primary care physician for a sports physical. She is a collegiate athlete planning on competing in basketball this winter. She denies fevers, chills, chest pain, palpitations, or shortness of breath. Family history includes anxiety in her mother, myocardial infarction in her father at age 50, and sudden cardiac death in her paternal cousin at age 17. Vital signs include a temperature of 37.0°C (98.6°F), respiratory rate of 12 breaths/min, heart rate of 72 beats/min, and blood pressure of 115/62 mmHg. The cardiac exam is notable for a crescendo-decrescendo systolic murmur best heard along the left sternal border. Faint bibasilar crackles are also present. There is no abdominal guarding or rebound tenderness. Her pulses are 2+ and biphasic. Which of the following is consistent with this patient’s most likely diagnosis?

A. Interventricular septal thickening

B. Left ventricular chamber dilatation

C. Concentric left ventricular hypertrophy

D. Left atrial chamber dilatation

E. Concentric left atrial hypertrophy



A!

reason:

This patient’s systolic murmur, bibasilar crackles, and a family history significant for sudden cardiac death in her paternal cousin are highly suggestive of hypertrophic cardiomyopathy (HCM). During exercise, affected patients may develop dyspnea and syncope. Diastolic dysfunction, due to ventricular hypertrophy, inhibits sufficient ventricular filling, leading to backpressure transmission to the pulmonary circulation. With progression of the disease, pulmonary edema may ensue. 


HCM is caused by a genetic mutation in sarcomere proteins (e.g., β-myosin heavy chain, myosin-binding protein C), resulting in marked, asymmetric septal-predominant left ventricular hypertrophy. It is often detected incidentally during routine physical exams, though some patients develop exercise-induced syncope. 60-70% of cases are familial, displaying an autosomal dominant inheritance pattern.


On auscultation, a crescendo-decrescendo systolic ejection murmur that intensifies with decreased preload (e.g., Valsalva, standing) indicates HCM. Systolic anterior motion (SAM) of the mitral valve can lead to inadequate closure, resulting in mitral valve regurgitation, sometimes causing a holosystolic murmur. This is explained by the Venturi effect.

500

A 62-year-old female with a past medical history of hypertension and hyperlipidemia presents to the cardiologist complaining of intermittent chest pain. The pain occurs when she walks, during times of stress, or from exposure to cold environments. The only alleviating factor is rest. Her current medications include chlorthalidone and simvastatin. Vital signs include a temperature of 36.9°C (98.4°F), respiratory rate of 14 breaths/min, heart rate of 76 beats/min, and blood pressure of 128/63 mmHg. Physical exam reveals an overweight woman in no acute distress. Her extraocular movements are intact. There is no tenderness over the chest wall. Lungs are clear to auscultation bilaterally. She has no hepatosplenomegaly or abdominal tenderness. Cardiac biomarkers are within normal limits. The patient is started on a medication that increases coronary blood vessel filling time. Which of the following regions is most affected by the hemodynamic changes caused by this medication?

A. Endocardium

B. Epicardium

C. Myocardium

D. Pericardium

E. Subendocardium



E! subendocardium


reason:

This patient is experiencing classic symptoms of stable angina, which is typically treated with medications such as beta-blockers, nitrates, or calcium channel blockers. These medications help increase coronary blood vessel filling time, thereby improving blood supply to the heart muscle. The subendocardium is the region of the heart most affected by these hemodynamic changes, as it is the innermost layer of the heart muscle and has the highest oxygen demand. The subendocardium is most vulnerable to ischemia during periods of increased oxygen demand or decreased coronary perfusion.